Thomas' Calculus 13th Edition

Published by Pearson
ISBN 10: 0-32187-896-5
ISBN 13: 978-0-32187-896-0

Chapter 1: Functions - Practice Exercises - Page 38: 65

Answer

$16.98m $

Work Step by Step

As shown in the figure, we are given $\bar{BC}=10$ m, BT angle to horizontal of $35^\circ,$ CT angle to horizontal of $50^\circ$. Assume the height of the pols is $ x $. We have: $\frac{x}{tan(35)}-\frac{x}{tan(50)}=10$, Solve for x: $ x=16.98m $
Update this answer!

You can help us out by revising, improving and updating this answer.

Update this answer

After you claim an answer you’ll have 24 hours to send in a draft. An editor will review the submission and either publish your submission or provide feedback.